0
$\begingroup$

I would like to know what the "Chow countability argument or HIlbert schemes countability argument" says in order to finish an exercise. Any reference will also be very useful :)!

$\endgroup$
2
  • 6
    $\begingroup$ Some context would be very useful for us to answer the question too. $\endgroup$ Commented Sep 27, 2021 at 14:33
  • 2
    $\begingroup$ I've never heard the phrase "Chow countability argument" before. Google seems confused as well. Where did you encounter it? (If I had to guess, it has something to do with the fact that Chow varieties/Hilbert schemes have countably many components. But this is just a guess.) $\endgroup$ Commented Sep 27, 2021 at 15:07

0

You must log in to answer this question.

Start asking to get answers

Find the answer to your question by asking.

Ask question

Explore related questions

See similar questions with these tags.